1answer.
Ask question
Login Signup
Ask question
All categories
  • English
  • Mathematics
  • Social Studies
  • Business
  • History
  • Health
  • Geography
  • Biology
  • Physics
  • Chemistry
  • Computers and Technology
  • Arts
  • World Languages
  • Spanish
  • French
  • German
  • Advanced Placement (AP)
  • SAT
  • Medicine
  • Law
  • Engineering
Bad White [126]
2 years ago
12

What is the greatest value of 2,463.9051

Mathematics
1 answer:
RoseWind [281]2 years ago
6 0

Hey There @Bre18016,

The answer is \boxed{2}

The greatest value of 2,463.9051 would be the thousands place (2) simply as it is the biggest number out of the other places.

For instance, if we had the number 300, 3 would be the greatest value.

Or let's say we had 10,000 the 1 would be the greatest value.

Furthermore, you could look at the first digit in the entire number to deter mine the greatest value.

You might be interested in
A ball is shot straight up into the air from the ground with initial velocity of 45ft/sec. Assuming that the air resistance can
Nesterboy [21]
V2^2 = V1^2 - 2gh 

<span>=> 0 = 44^2 - 2 (32.2) h </span>

<span>=> h = 44^2 / 64.4 = 30.062 ft</span>
6 0
3 years ago
Tyler is offered to choose between three boxes that look identical. The first box contains 1 red pills and 19 blue pills, the se
Dafna1 [17]

Answer:

A) 0.15

B) 0.36

Step-by-step explanation:

First Box consist 1 red pills and 19 blue pills

second box consist 6 red pills and 14 blue pills

Third box consist 2 red pills and 18 blue pills

a) Prob ( randomly chosen pill is red) = \frac{1}{3} \times \frac{1}{20} + \frac{1}{3} \times \frac{6}{20} + \frac{1}{3} \times \frac{2}{20}

\frac{1}{60} (1+6+2) = 0.15

b)Prob ( 1 reds and 10 blue pilss) =\frac{1}{3} \frac{^1C_1\times ^{19}C_{10}}{^{20}C_{11}} + \frac{1}{3} \frac{^6C_1\times ^{14}C_{10}}{^{20}C_{11}} + \frac{1}{3} \frac{^2C_1\times ^{18}C_{10}}{^{20}C_{11}}

= \frac{1}{3} 1.106 = 0.368

7 0
3 years ago
How many feet are there in 216 yards
Dafna11 [192]
There are 648 ft in 216 yds. :)
8 0
3 years ago
You are given the
r-ruslan [8.4K]

Answer:

1.→n=15

2.→n=0

Step-by-step explanation:

The two expressions given are

⇒A number is said to be rational if it can be expressed in the form of , where ,q≠0.And it's decimal expansion will be either terminating or non terminating repeating.

We have to find smallest value of n, for which each of these two expressions will be rational.

So, 1.→n=15

and 2.→n=0

Have a nice day! -Alpha

Step-by-step explanation: ;)

8 0
3 years ago
Do you agree? explain why or why not
castortr0y [4]
No. when plugging 4 into the equation, 0 is not the final result. instead it is 6. 

square root of 2(4)+1 = square root of 9 = 3 

3+3 = 6
and 6 is not 0 so x cannot = 4

3 0
3 years ago
Read 2 more answers
Other questions:
  • What is 16% of 375?Thank You
    11·1 answer
  • Ron draws 16 cards from a standard deck of 52. The deck is made up of equal numbers of four suits: clubs,diamonds, hearts, and s
    9·1 answer
  • Isnt this skewed left since majority of the data is on the right side.
    7·1 answer
  • What is 3 2/8 times 4 1/5
    9·1 answer
  • Find all solutions to (X+5)(2x-3)=0
    7·1 answer
  • Find the product of 98 and 863
    15·2 answers
  • Suppose that a pregnancy test kit, while tested on women, produces 95% true positive and 90% true negative. Suppose that 7% of w
    14·1 answer
  • Do y'all understand this question if so plz help ASAP....
    13·1 answer
  • Puntuación de una Línea determinada?
    10·1 answer
  • Triangles ABC and DEF are similar. Find e. <br><br> A.) 11<br> B.) 12<br> C.) 10<br> D.) 9
    14·2 answers
Add answer
Login
Not registered? Fast signup
Signup
Login Signup
Ask question!